Linear Algebra with Proof by Contradiction

Click For Summary
The discussion centers on proving that if the union of two subspaces of a vector space V is itself a subspace, then one subspace must be contained within the other. The initial assumption is that neither subspace is contained in the other, leading to a contradiction when considering elements from both subspaces. The proof involves demonstrating that if both subspaces are not contained within each other, it results in contradictions regarding their sums. The conclusion drawn is that at least one of the initial assumptions must be false, reinforcing the requirement for one subspace to be contained in the other. The importance of understanding Conditional Proof and sub-proof structures in such proofs is emphasized for clarity and control.
Devil Moo
Messages
44
Reaction score
1
This is a linear algebra question which I am confused.

1. Homework Statement


Prove that "if the union of two subspaces of ##V## is a subspace of ##V##, then one of the subspaces is contained in the other".

The Attempt at a Solution



Suppose ##U##, ##W## are subspaces of ##V##. ##U \cup W## is a subspace of ##V##. (statement A)

Suppose ##U## is not contained in ##W## and vice versa. (statement B)

Let ##u \in U, \not\in W## and ##w \in W, \not\in U##.

##u \in U \cup W, w \in U \cup W##
##u + w \in U \cup W##
##u + w \in U## or ##u + w \in W##

Suppose ##u + w \in U## (statement C)

##u + w + (-u) \in U##
##w \in U##
It leads to contradiction.

Which one does it conclude? (if A is true, then B is false) or (if A and B are true, then C is false)
 
Physics news on Phys.org
It's the second one: (if A and B are true, then C is false)

Since you have proven C false, you can conclude that ##u+v\in W##. By similar steps to before you can also conclude that that is false.

You can then, with a few more steps, conclude that at least one of A or B must be false.

Arguments of this kind are much easier to understand and control if you grasp the notion of Conditional Proof or sub-proof. Every time you make a new assumption, you are opening a new sub-proof. When you reach a contradiction, you close that sub-proof, concluding the opposite of the assumption that you used to open it. It is not unusual to have several nested levels of proof. In your case there are three levels.
 
For the proof by contradiction about ##\sqrt 2## is a irrational number, we conclude that it is true once we find the contradiction.
In this case, why can't I conclude that ##u + v \in W## is true?

Is ##not (u + v \in U) = u + v \in W## wrong?
 
Last edited:
Devil Moo said:
In this case, why can't I conclude that ##u + v \in W## is true?
You can. Read the second line of my post again.

Then you go on to prove that it is also false, which is a second contradiction, which tells you that either A or B must be false.
 
Question: A clock's minute hand has length 4 and its hour hand has length 3. What is the distance between the tips at the moment when it is increasing most rapidly?(Putnam Exam Question) Answer: Making assumption that both the hands moves at constant angular velocities, the answer is ## \sqrt{7} .## But don't you think this assumption is somewhat doubtful and wrong?

Similar threads

  • · Replies 1 ·
Replies
1
Views
2K
Replies
1
Views
1K
  • · Replies 4 ·
Replies
4
Views
2K
Replies
8
Views
2K
  • · Replies 5 ·
Replies
5
Views
2K
  • · Replies 4 ·
Replies
4
Views
3K
  • · Replies 1 ·
Replies
1
Views
1K
  • · Replies 10 ·
Replies
10
Views
2K
  • · Replies 7 ·
Replies
7
Views
1K
  • · Replies 3 ·
Replies
3
Views
3K